Let $ a$, $ b$, $ c$ be positive real numbers such that $ abc=1$. Prove that $ \frac{1}{1+b+c}+\frac{1}{1+c+a}+\frac{1}{1+a+b}\leq\frac{1}{2+a}+\frac{1}{2+b}+\frac{1}{2+c}$.
Problem
Source: Bulgaria 1997; Old and New Inequalities (both times the proposed solution is brute force)
Tags: inequalities, function, inequalities unsolved
08.08.2003 11:12
OKay.. cause today is my birthday and I promissed myself that at least on my birthday I will stop solving math problems.. I will just ask you if you used the oldie but goldie trick a=x/y, b=z/x and c=y/z because their product will be 1. hope that this is how is made! (I am not solving anything today )
08.08.2003 14:47
1st of all happy birthday Lagrangia! sencond of all - this inequality was given at a Romanian Test for 8-th graders (last school class before entering high-school in Romania), and obviously no-one did it. I've tried it then, using all elementary (and nice) ideas I could possible think of. Didn't work. Last year I found out that it was also used in a Bulgarian TST, and I succeded in solving it using the "brute force" (which they actually have in their original solution). Besides that, like that problem from Iran 1996, it is very unlikey that one can find a nice solution ...
08.08.2003 22:04
For the trick I prefer a=x^2/yz, b=y^2/zx, c=z^2/xy instead since it doesn't cause the loss of symmetry..
20.08.2004 11:28
Well,we can use a well-known method here.Take $f(a,b,c)=RHS-LHS$. Assume that $c=max \lbrace{a,b,c}\rbrace$ then we have: f(a,b,c)-f( \frac{a+b}{2}, \frac{a+b}{2},c)= (a-b)^2(\frac{1}{(2+a)(2+b)(4+a+b)}-\frac{1}{(1+a+c)(1+b+c)(2+2c+a+b)}) \geq 0 since c \geq 1 Now,prove it for f(a,a,b).I have expanded it and this is really disgusting . However,the last result is (a-1)^2(2a^4+9a^3+9a^2+4a+3) \geq 0.
12.09.2004 23:23
I just wanted to mention the appearance of the inequality with solution in two sources: 1. Titu Andreescu, Vasile Cîrtoaje, Gabriel Dospinescu, Mircea Lascu, Old and New Inequalities, Zalau: GIL 2004, problem 99. 2. American Mathematics Competitions: Mathematical Olympiads 1997-1998: Olympiad Problems from Around the World, Bulgaria 21, p. 23. Both solutions are almost the same: Brute force. The inequality doesn't seem to have a better proof. The second source can be freely downloaded from http://www.unl.edu/amc/a-activities/a4-for-students/problemtext/s-index.html EDIT: Since this website does not work properly, here are direct links to the files at the site: http://www.unl.edu/amc/a-activities/a4-for-students/problemtext/ http://www.unl.edu/amc/a-activities/a4-for-students/problemtext/geom-080399.ps http://www.unl.edu/amc/a-activities/a4-for-students/problemtext/geom-080399.tex http://www.unl.edu/amc/a-activities/a4-for-students/problemtext/ineqs-080299.ps http://www.unl.edu/amc/a-activities/a4-for-students/problemtext/ineqs-080299.tex http://www.unl.edu/amc/a-activities/a4-for-students/problemtext/mc96-97-01feb.dvi http://www.unl.edu/amc/a-activities/a4-for-students/problemtext/mc96-97-01feb.pdf http://www.unl.edu/amc/a-activities/a4-for-students/problemtext/mc96-97-01feb.ps http://www.unl.edu/amc/a-activities/a4-for-students/problemtext/mc97-98-01feb.dvi http://www.unl.edu/amc/a-activities/a4-for-students/problemtext/mc97-98-01feb.pdf http://www.unl.edu/amc/a-activities/a4-for-students/problemtext/mc97-98-01feb.ps Darij
13.09.2004 22:29
FOR several months I am trying to prove the general case of this damn inequality, but I did not succeded. Did you?
09.03.2005 03:36
It seem be post some times in mathlinks. And when you Multi two hand side,it's very clearly,not difficult. I had solve it before by this way. Other way,It can be prove immediate by the generel: $\sum\frac{1}{n-1+x_i}\le 1$ if $\prod{x_i}=1$ And then: $\sum\frac{1}{n-1+x_i} \le 1 \le \sum\frac{1}{1+(n-1)x_i}$
09.03.2005 13:48
But $1 \ge \sum{\frac{1}{n-1+x_i}}$.
10.03.2005 00:36
hungkhtn wrote: It seem be post some times in mathlinks. $\sum\frac{1}{n-1+x_i}\le 1$ if $\prod{x_i}=1$ And then: $\sum\frac{1}{n-1+x_i} \le 1 \le \sum\frac{1}{1+(n-1)x_i}$ Here is a proof for the first one. $f(x_{1},x_{2},...,x_{n})=\sum\frac{1}{n-1+x_i}$.As $\prod{x_i}=1$ we may assume $x_{1}\ge{1}, x_{2}\le{1}$. We shall prove that $f(x_{1},x_{2},...,x_{n})\le{f(1,x_{1}x_{2},...,x_{n})}$. And this is true because after a little computation we obtain $(1-x_{1})(x_{2}-1)(x_{1}x_{2}+(n-1)^{2})\ge{0}$ which is obviously true. So we have $f(x_{1},x_{2},...,x_{n})\le{f(1,x_{1}x_{2},...,x_{n})}\le ...\le{f(1,...,1)}=1$. QED
24.05.2005 23:33
i have just solved by clearing denominators + muirhead symmetric inequality. but is there a short solution???????
25.05.2005 12:50
i'm very sure that mixing variables works, but i'm not sure if it's short.
25.05.2005 13:46
Here is solution :asumme a\geq b\geq c R.H.S-L.H.S=\sum \frac{b-1}{(2+a)(1+a+b)}\geq \frac{a+b+c-3}{(2+a)(1+a+b)}\geq 0
01.07.2005 16:45
keira_khtn wrote: $\sum \frac{b-1}{(2+a)(1+a+b)}\geq \frac{a+b+c-3}{(2+a)(1+a+b)}$ This inequality is not trivial at all, because $\frac{c-1}{(2+b)(1+b+c)}$ is negative and can't simply change the denominator.
01.07.2005 18:07
this is from bulgaria ninety something i guess the best solution is bruteforce put : x=a+b+c and y=ab+ac+bc and express each side in terms of these variables and then pure calculation till you get this very ugly inequality wich is true using AM-GM
03.07.2005 16:00
Bojan Basic wrote: keira_khtn wrote: $\sum \frac{b-1}{(2+a)(1+a+b)}\geq \frac{a+b+c-3}{(2+a)(1+a+b)}$ This inequality is not trivial at all, because $\frac{c-1}{(2+b)(1+b+c)}$ is negative and can't simply change the denominator. Oh,i'm false..
29.07.2005 02:57
i m not sure whether i m right, i found a very easy solution though look, use the classical technic , put $ a = \frac xy , b = \frac yz , c = \frac zx $ and assume that $ x \geq y \geq z $ then we have $ \sum \frac {1}{1+a+b} = \sum \frac {yz}{yz + zx + y^2} \leq \sum \frac {yz}{2yz + zx} $ since $ y^2 \geq yz $ then $ \sum \frac {yz}{2yz + zx} = \sum \frac {1}{2 + \frac xy} = \sum \frac {1}{2 + a} $ hence $ \sum \frac {1}{1+a+b} \leq \sum \frac {1}{2 + a} $ any mistake??
29.07.2005 03:01
Al-Khawarizmi wrote: $\sum \frac {yz}{yz + zx + y^2} \leq \sum \frac {yz}{2yz + zx} $ since $ y^2 \geq yz $ any mistake?? Yes. We do have $y^2 \geq yz$ but the other, similar inequalities do not necessarily hold.
29.07.2005 03:04
my solution is wrong, sorry!
29.07.2005 03:12
thank you Arne, i did not see your post, you are so fast here people
10.01.2006 15:13
I think the best solution is to use the fact that $\frac{1}{2a+1}+\frac{1}{2b+1}\geq\frac{2}{a+b+1}$
10.01.2006 22:57
[EDIT] I applyed Jensen's incorrectly. It is funny how you get the desired result with the stupid sign mistake
10.01.2006 23:02
Guys, you know this is a hard problem and the only known solution is brute-force, so if you think you have a short simple proof, why can't you bother check it twice before posting it? For instance, Silouan's inequality silouan wrote: $\frac{1}{2a+1}+\frac{1}{2b+1}\geq\frac{2}{a+b+1}$ is true, but doesn't help solving the initial problem. Yassin's Ezbakhe Yassin wrote: $1\leq \frac{1}{2+a}+\frac{1}{2+b}+\frac{1}{2+c}$ is true, but with $\geq$ instead of $\leq$, so it isn't too useful either. Sorry, but I just wanted to tell the truth... Darij
10.01.2006 23:03
Ezbakhe Yassin wrote: Here is a more direct form. Hope it is correct. $\frac{1}{1+a+b}+\frac{1}{1+b+c}+\frac{1}{1+c+a}\leq \frac{3}{\frac1{3}(1+a+b+1+b+c+1+c+a)}$$=\frac9{3+2a+2b+2c}\leq 1$ $\frac{1}{1+a+b}+\frac{1}{1+b+c}+\frac{1}{1+c+a}\geq \frac9{3+2a+2b+2c}$ by Cauchy
11.01.2006 09:50
silouan wrote: I think the best solution is to use the fact that $\frac{1}{2a+1}+\frac{1}{2b+1}\geq\frac{2}{a+b+1}$ Then We must prove $\frac{1}{a+2}+\frac{1}{b+2}+\frac{1}{c+2}\geq \frac{1}{2a+1}+\frac{1}{2b+1}+\frac{1}{2c+1}$ But it was wrong
09.10.2007 06:44
alekk wrote: Let $ a$, $ b$, $ c$ be positive real numbers such that $ abc = 1$. Prove that $ \frac {1}{1 + b + c} + \frac {1}{1 + c + a} + \frac {1}{1 + a + b}\leq\frac {1}{2 + a} + \frac {1}{2 + b} + \frac {1}{2 + c}$. Let $ x = a + b + c;y = ab + bc + ca$ Inequality becomes: $ \frac {x^2 + 4x + y + 3}{x^2 + 2x + y + xy} \leq \frac {12 + 4x + y}{9 + 4x + 2y}$ <=>$ \frac {2x + 3 - xy}{x^2 + 2x + y + xy} \leq \frac {3 - y}{9 + 4x + 2y}$ Note that $ x\geq3;y \geq 3;x^2 \geq 3y$ so we have $ \frac {5}{3}x^2y\geq 5x^2;\frac {x^2y}{3}\geq y^2;xy^2 \geq 9x;5xy \geq 15x;xy \geq 3y;x^2y \geq 27$ Done!
12.10.2016 17:34
alekk wrote: Let $ a$, $ b$, $ c$ be positive real numbers such that $ abc=1$. Prove that $ \frac{1}{1+b+c}+\frac{1}{1+c+a}+\frac{1}{1+a+b}\leq\frac{1}{2+a}+\frac{1}{2+b}+\frac{1}{2+c}$. Let $ a$, $ b$, $ c$, $ d$ be positive real numbers such that $ abcd=1$.Prove that $$ \frac{1}{1+b+c+d}+\frac{1}{1+c+d+a}+\frac{1}{1+d+a+b}+\frac{1}{1+a+b+c} \le\frac{1}{3+a+} +\frac{1}{3+b} + \frac{1}{3+c} + \frac{1}{3+d} $$
12.05.2020 05:08
alekk wrote: Let $ a$, $ b$, $ c$ be positive real numbers such that $ abc=1$. Prove that $$ \frac{1}{1+b+c}+\frac{1}{1+c+a}+\frac{1}{1+a+b}\leq\frac{1}{2+a}+\frac{1}{2+b}+\frac{1}{2+c}$$ Let $ a$, $ b$, $ c$ be positive real numbers such that $ abc=1$.Prove that $$\frac{1}{3}\left(\frac{1}{\sqrt{2+a}} +\frac{1}{\sqrt{2+b}}+\frac{1}{\sqrt{2+c}}\right)^2\leq \frac{1}{2+a}+\frac{1}{2+b}+\frac{1}{2+c} \leq 1$$$$\frac{1}{3+a}+\frac{1}{3+b}+\frac{1}{3+c}\le \frac{3}{4}$$here here
07.04.2021 15:41
https://artofproblemsolving.com/community/c6h1655438p10498091: Let $a,b,c>0$ and $abc=1$. Show that $$\frac{a}{b+c+1}+\frac{b}{c+a+1}+\frac{c}{a+b+1} \geqslant \frac{a}{a+2}+\frac{b}{b+2}+\frac{c}{c+2}\ge 1$$Let $a,b,c>0$ and $abc=1$. Show that $$\frac{1}{b+c+1}+ \frac{1}{c+a+1}+ \frac{1}{a+b+1}\leq\frac{1}{a+2}+\frac{1}{b+2}+\frac{1}{c+2}\leq 1.$$ sqing wrote: Let $ a$, $ b$, $ c$, $ d$ be positive real numbers such that $ abcd=1$.Prove that $$ \frac{1}{1+b+c+d}+\frac{1}{1+c+d+a}+\frac{1}{1+d+a+b}+\frac{1}{1+a+b+c} \le\frac{1}{3+a} +\frac{1}{3+b} + \frac{1}{3+c} + \frac{1}{3+d} $$ here